matheraum.de
Raum für Mathematik
Offene Informations- und Nachhilfegemeinschaft

Für Schüler, Studenten, Lehrer, Mathematik-Interessierte.
Hallo Gast!einloggen | registrieren ]
Startseite · Forum · Wissen · Kurse · Mitglieder · Team · Impressum
Forenbaum
^ Forenbaum
Status Schulmathe
  Status Primarstufe
  Status Mathe Klassen 5-7
  Status Mathe Klassen 8-10
  Status Oberstufenmathe
    Status Schul-Analysis
    Status Lin. Algebra/Vektor
    Status Stochastik
    Status Abivorbereitung
  Status Mathe-Wettbewerbe
    Status Bundeswettb. Mathe
    Status Deutsche MO
    Status Internationale MO
    Status MO andere Länder
    Status Känguru
  Status Sonstiges

Gezeigt werden alle Foren bis zur Tiefe 2

Navigation
 Startseite...
 Neuerdings beta neu
 Forum...
 vorwissen...
 vorkurse...
 Werkzeuge...
 Nachhilfevermittlung beta...
 Online-Spiele beta
 Suchen
 Verein...
 Impressum
Das Projekt
Server und Internetanbindung werden durch Spenden finanziert.
Organisiert wird das Projekt von unserem Koordinatorenteam.
Hunderte Mitglieder helfen ehrenamtlich in unseren moderierten Foren.
Anbieter der Seite ist der gemeinnützige Verein "Vorhilfe.de e.V.".
Partnerseiten
Weitere Fächer:

Open Source FunktionenplotterFunkyPlot: Kostenloser und quelloffener Funktionenplotter für Linux und andere Betriebssysteme
StartseiteMatheForenLineare Algebra - Moduln und VektorräumeSumme Unterräume II
Foren für weitere Schulfächer findest Du auf www.vorhilfe.de z.B. Informatik • Physik • Technik • Biologie • Chemie
Forum "Lineare Algebra - Moduln und Vektorräume" - Summe Unterräume II
Summe Unterräume II < Moduln/Vektorraum < Lineare Algebra < Hochschule < Mathe < Vorhilfe
Ansicht: [ geschachtelt ] | ^ Forum "Lineare Algebra - Moduln und Vektorräume"  | ^^ Alle Foren  | ^ Forenbaum  | Materialien

Summe Unterräume II: Tipp
Status: (Frage) beantwortet Status 
Datum: 10:42 Fr 09.01.2015
Autor: Ne0the0ne

Aufgabe
Zeigen Sie: Für drei (endlichdimensionale) Unterräume [mm] U_{i} [/mm] eines Vektorraumes V gilt:
4 dim [mm] U_{1} [/mm] + 4 dim [mm] U_{2} [/mm] + 4 dim [mm] U_{3} [/mm] = dim [mm] ((U_{1} \cap U_{2})+U_{3}) [/mm] + dim [mm] ((U_{2} \cap U_{3})+U_{1}) [/mm] + dim [mm] ((U_{3} \cap U_{1})+U_{2}) [/mm] + dim [mm] ((U_{1}+U_{2})\cap U_{3}) [/mm] + dim [mm] ((U_{2}+U_{3})\cap U_{1}) [/mm] + dim [mm] ((U_{3}+U_{1})\cap U_{2}) [/mm] + 3dim [mm] (U_{1} [/mm] + [mm] U_{2} [/mm] + [mm] U_{3}) [/mm] + 3dim [mm] (U_{1} \cap U_{2} \cap U_{3}). [/mm]

Mein Ansatz:
Aus 4 dim [mm] U_{1} [/mm] + 4 dim [mm] U_{2} [/mm] + 4 dim [mm] U_{3} [/mm] habe ich erstmal die 4 und dim ausgeklammert, also 4dim [mm] (U_{1} [/mm] + [mm] U_{2} [/mm] + [mm] U_{3}). [/mm]
Die entstandene Klammer habe ich dann umgeschrieben in dim [mm] ((U_{1} [/mm] + [mm] U_{2}) [/mm] + [mm] U_{3})+dim (U_{1} [/mm] + [mm] (U_{2} [/mm] + [mm] U_{3})) [/mm] + dim [mm] ((U_{1} [/mm] + [mm] U_{3}) [/mm] + [mm] U_{2}) [/mm] + dim [mm] (U_{1} [/mm] + [mm] U_{2} [/mm] + [mm] U_{3}). [/mm]

Jetzt wollte ich auf die einzelnen Summanden den Dimensionssatz anwenden, der besagt, dass dim(U+W)=dim U + dim W - [mm] dim(U\cap [/mm] W) ist.

Allerdings habe ich jetzt Probleme den 3. Unterraum in der Formel unterzubringen.
Darum würde ich gerne dafür einen Tipp bekommen bzw. wissen, ob mein Ansatz schon mal in die richtige Richtung geht.

MfG Ne0the0ne

        
Bezug
Summe Unterräume II: Antwort
Status: (Antwort) fertig Status 
Datum: 10:58 Fr 09.01.2015
Autor: fred97


> Zeigen Sie: Für drei (endlichdimensionale) Unterräume
> [mm]U_{i}[/mm] eines Vektorraumes V gilt:
>  4 dim [mm]U_{1}[/mm] + 4 dim [mm]U_{2}[/mm] + 4 dim [mm]U_{3}[/mm] = dim [mm]((U_{1} \cap U_{2})+U_{3})[/mm]
> + dim [mm]((U_{2} \cap U_{3})+U_{1})[/mm] + dim [mm]((U_{3} \cap U_{1})+U_{2})[/mm]
> + dim [mm]((U_{1}+U_{2})\cap U_{3})[/mm] + dim [mm]((U_{2}+U_{3})\cap U_{1})[/mm]
> + dim [mm]((U_{3}+U_{1})\cap U_{2})[/mm] + 3dim [mm](U_{1}[/mm] + [mm]U_{2}[/mm] +
> [mm]U_{3})[/mm] + 3dim [mm](U_{1} \cap U_{2} \cap U_{3}).[/mm]
>  Mein Ansatz:
>  Aus 4 dim [mm]U_{1}[/mm] + 4 dim [mm]U_{2}[/mm] + 4 dim [mm]U_{3}[/mm] habe ich
> erstmal die 4 und dim ausgeklammert, also 4dim [mm](U_{1}[/mm] +
> [mm]U_{2}[/mm] + [mm]U_{3}).[/mm]


Das kannst Du doch nicht machen !!!

Stell Dir vor, es wäre [mm] U_1=U_2=U_3 [/mm] und dim [mm] U_1=1. [/mm]

Dann ist  [mm] U_1+U_2+U_3 [/mm] = [mm] U_1. [/mm]

Nach Deiner "Rechnung" hätten wir:

    12=4.

FRED

>  Die entstandene Klammer habe ich dann umgeschrieben in dim
> [mm]((U_{1}[/mm] + [mm]U_{2})[/mm] + [mm]U_{3})+dim (U_{1}[/mm] + [mm](U_{2}[/mm] + [mm]U_{3}))[/mm] +
> dim [mm]((U_{1}[/mm] + [mm]U_{3})[/mm] + [mm]U_{2})[/mm] + dim [mm](U_{1}[/mm] + [mm]U_{2}[/mm] +
> [mm]U_{3}).[/mm]
>  
> Jetzt wollte ich auf die einzelnen Summanden den
> Dimensionssatz anwenden, der besagt, dass dim(U+W)=dim U +
> dim W - [mm]dim(U\cap[/mm] W) ist.
>  
> Allerdings habe ich jetzt Probleme den 3. Unterraum in der
> Formel unterzubringen.
>  Darum würde ich gerne dafür einen Tipp bekommen bzw.
> wissen, ob mein Ansatz schon mal in die richtige Richtung
> geht.
>  
> MfG Ne0the0ne


Bezug
        
Bezug
Summe Unterräume II: Rückfrage
Status: (Frage) beantwortet Status 
Datum: 11:50 Fr 09.01.2015
Autor: Ne0the0ne

Vielen Dank schon mal für die Korrigierung.
Ich stehe immer noch an dem Punkt der Orientierungslosigkeit.
Ich weiß nur, dass ich den Dimensionssatz anwenden muss.

Anbei habe ich bei Recherchen diese Formel gefunden:
dim U + dim V + dim W = dim (U+V+W) + dim ((U+V) [mm] \cap [/mm] W) + dim (U [mm] \cap [/mm] V)

Allerdings habe ich hier das Problem mit der Umsetzung der Formel aufgrund der vierfachen Dimension (lt. Aufgabenstellung).

Bezug
                
Bezug
Summe Unterräume II: Antwort
Status: (Antwort) fertig Status 
Datum: 14:53 Fr 09.01.2015
Autor: angela.h.b.


> Vielen Dank schon mal für die Korrigierung.
>  Ich stehe immer noch an dem Punkt der
> Orientierungslosigkeit.
>  Ich weiß nur, dass ich den Dimensionssatz anwenden muss.

Hallo,

ja, so ist es.

>  
> Anbei habe ich bei Recherchen diese Formel gefunden:
> dim U + dim V + dim W = dim (U+V+W) + dim ((U+V) [mm]\cap[/mm] W) +
> dim (U [mm]\cap[/mm] V)

Hui, das war sicher nicht gleich der allererste Treffer.

Normalerweise lernt man den Dimensionssatz so: sind A,B Unterräume von V, so gilt
[mm] dim(A+B)=dimA+dimB-dim(A\cap [/mm] B).
So wird es sicher auch in der VL drangewesen sein.

Mit dem Wissen, daß für Unterräume U,V,W eines VRes V'
U+V ein Unterraum ist,
bekommst Du die Formel, die Du gefunden hast.
Rechne das ruhig mal nach - es scheint mir für Deine Aufgabenstellung nicht ganz unnütz zu sein...


Ich würde bei Deiner Aufgabe mit der rechten Seite der Gleichung beginnen, und so lange weitermachen, bis die linke dasteht. Schau:

[mm] \red{dim (\underbrace{(U_{1} \cap U_{2})}_{A}+\underbrace{U_{3}}_{B})} [/mm] + dim [mm] ((U_{2} \cap U_{3})+U_{1}) [/mm] + dim [mm] ((U_{3} \cap U_{1})+U_{2}) [/mm] + dim [mm] ((U_{1}+U_{2})\cap U_{3}) [/mm] + [mm] dim((U_{2}+U_{3})\cap U_{1}) [/mm] + dim [mm] ((U_{3}+U_{1})\cap U_{2}) [/mm] + 3dim [mm] (U_{1} [/mm] + [mm] U_{2} [/mm] + [mm] U_{3}) [/mm] + 3dim [mm] (U_{1} \cap U_{2} \cap U_{3}) [/mm]

[mm] =\red{dim (U_{1} \cap U_{2})+dimU_{3}-dim(U_1\cap U_2\cap U_3)} [/mm] + dim [mm] ((U_{2} \cap U_{3})+U_{1}) [/mm] + dim [mm] ((U_{3} \cap U_{1})+U_{2}) [/mm] + dim [mm] ((U_{1}+U_{2})\cap U_{3}) [/mm] + dim [mm] ((U_{2}+U_{3})\cap U_{1}) [/mm] + dim [mm] ((U_{3}+U_{1})\cap U_{2}) [/mm] + 3dim [mm] (U_{1} [/mm] + [mm] U_{2} [/mm] + [mm] U_{3}) [/mm] + 3dim [mm] (U_{1} \cap U_{2} \cap U_{3}) [/mm]

=dim [mm] (U_{1} \cap U_{2})+dimU_{3}+ [/mm] dim [mm] ((U_{2} \cap U_{3})+U_{1}) [/mm] + dim [mm] ((U_{3} \cap U_{1})+U_{2}) [/mm] + dim [mm] ((U_{1}+U_{2})\cap U_{3}) [/mm] + dim [mm] ((U_{2}+U_{3})\cap U_{1}) [/mm] + dim [mm] ((U_{3}+U_{1})\cap U_{2}) [/mm] + 3dim [mm] (U_{1} [/mm] + [mm] U_{2} [/mm] + [mm] U_{3}) [/mm] + 2dim [mm] (U_{1} \cap U_{2} \cap U_{3}), [/mm]

und dann so in diesem Stile weiterfrickeln.

LG Angela





>  
> Allerdings habe ich hier das Problem mit der Umsetzung der
> Formel aufgrund der vierfachen Dimension (lt.
> Aufgabenstellung).


Bezug
                        
Bezug
Summe Unterräume II: Danke
Status: (Mitteilung) Reaktion unnötig Status 
Datum: 11:01 Sa 10.01.2015
Autor: Ne0the0ne

Vielen, vielen lieben Dank für deine Hilfe.
Konnte endlich die Aufgabe lösen.

Habe durch deine Hilfestellung verstanden, dass die Summanden in der Dimensionsformel auch schon Durchschnittsmengen sein können! (Oh man, ich komme mir vor wie ein Anfänger)

Die "Formel" für 3 UVR stellte sich in der Tat als hilfreich heraus.


Bezug
Ansicht: [ geschachtelt ] | ^ Forum "Lineare Algebra - Moduln und Vektorräume"  | ^^ Alle Foren  | ^ Forenbaum  | Materialien


^ Seitenanfang ^
www.schulmatheforum.de
[ Startseite | Forum | Wissen | Kurse | Mitglieder | Team | Impressum ]